LSAT and Law School Admissions Forum

Get expert LSAT preparation and law school admissions advice from PowerScore Test Preparation.

 SLF
  • Posts: 40
  • Joined: Oct 01, 2013
|
#12665
Answer choice E seemed to me to be the big assumption...in that the mayor was assuming a daily charge would reduce traffic congestion...which was one of two objectives.

However, the answer key asserts that answer choice A is the correct answer.

When I was working the problem, I summarily dismissed answer choice A because I interpreted it as what the book calls an "opposite answer".

The stimulus specifically stated: "the system will not be ready until the end of next year."

So, an answer choice A that stated: "The mayor's plan to charge for driving downtown will be implemented before the end of next year." struck me as refuting something the stimulus said...hence my "jump" to the conclusion that it was an "opposite" answer choice and therefore incorrect.

Now that I know the correct answer, I can see that the stimulus is talking about what will happen as and after the plan is implemented...and so I can now see that one assumption is about whether the plan will be implemented at all.

So, any tips on how to more accurately identify the correct answer in a question like this one...and on how to NOT get thrown off track by seemingly opposite/conflicting phrases like the one's above?

It may be that the only answer is more practice and more careful reading...but as I was speeding along trying to be mindful of the elapsing time, I missed it. So, if you have anything that can help me, I would love to hear it.
User avatar
 KelseyWoods
PowerScore Staff
  • PowerScore Staff
  • Posts: 1079
  • Joined: Jun 26, 2013
|
#12677
Hi SLF,

Thanks for your question!

So one thing you need to be sure of in questions with arguments is to focus on the conclusion. It seems like you got caught up in the mayor's argument that charging for driving downtown will reduce congestion. But the actual conclusion of the author of this stimulus is that last sentence, after the "Therefore": when the plan is implemented, payment won't be effectively enforced.

Also, you are correct that the stimulus says that "this system will not be ready until the end of next year." But you need to make sure you're clear on the "system" that they are referring to. The "system" is the digital cameras and computerized automobile registration that will be used to enforce the plan, NOT the plan itself. Answer choice (A) doesn't conflict with that statement because the plan to charge for driving and the enforcement system are two different things.

Basically, the author has said that since the enforcement system won't be ready until the end of next year, the plan won't be effectively enforced at first. So he's assuming (as answer choice (A) says) that the plan requiring people to pay for driving downtown will go into effect sometime before they have the technology in place to actually hold people accountable for paying.

If you use the Assumption Negation Technique on (A) it would read: "The mayor's plan will NOT go into effect before the end of next year" or "The mayor's plan will go into effect AFTER the end of next year." However you negate it, the negated statement attacks the conclusion. If the mayor's plan isn't implemented until AFTER the technology is in place to enforce it, that directly attacks the conclusion that the plan won't be enforced when it's first implemented.

It's good that you're taking the time to really analyze your practice test questions and the way you think through them! The lesson to learn from this question is to identify and focus on the main conclusion of the stimulus and to read carefully to make sure when you have vague nouns (like "system") you know exactly what they are referring to.

Hope that helps!

Best,
Kelsey
 oli_oops
  • Posts: 37
  • Joined: Aug 22, 2018
|
#62959
Hi,

Can someone explain why B is incorrect?
I get why A is correct.

Thank you!!
 Robert Carroll
PowerScore Staff
  • PowerScore Staff
  • Posts: 1787
  • Joined: Dec 06, 2013
|
#62964
oli,

There is no need to assume that the city will incur a budget deficit. They'll be missing out on some revenue. Will it matter? Will the amount cause a budget shortfall? They might have such a budget surplus right now that missing out on some revenue for a while won't affect anything. The assumption here requires me to know some very specific and unmentioned stuff about the budget, and thus it's not required.

Robert Carroll
User avatar
 irelandbryant
  • Posts: 2
  • Joined: Jun 21, 2023
|
#102171
Can someone explain why C is wrong?
 Luke Haqq
PowerScore Staff
  • PowerScore Staff
  • Posts: 742
  • Joined: Apr 26, 2012
|
#102199
Hi irelandbryant!

To understand this question, let's start with the conclusion. This is the last sentence of the stimulus: "Therefore, when the mayor’s plan is first implemented, payment of the charge will not be effectively enforced."

Why does the author conclude this? The author reasons that the computerized system will doing the enforcement, but the system won't be ready until next year. Nothing in the stimulus, however, tells us when exactly the mayor's plan will first be implemented. If the plan is first implemented in a year, or in two years, then the conclusion doesn't follow. In that length of time, the computerized system will already be up and running.

Applying the Assumption Negation technique to answer (A) reflects this reasoning. Answer choice (A) states, "The mayor’s plan to charge for driving downtown will be implemented before the end of next year." The Assumption Negation technique involves negating the answer choice and then plugging that back into the stimulus; if it makes the argument fall apart/weakens it, then we know it's an assumption on which the argument depends. A negation of (A) would be "The mayor’s plan to charge for driving downtown will [not] be implemented before the end of next year." If that were true, the argument would fall apart that payment would not be effectively enforced when the plan is first implemented. If it's not first implemented until the end of next year, the computerized system will be up and running by then.

The negation of answer choice (C) would be "The plan to charge for driving downtown should [not] be implemented as soon as payment of the charge can be effectively enforced." It's not clear how this would make the argument fall apart. In addition, the word "should" in the answer choice should raise a flag. Often, a stimulus will have several descriptive statements, and then a normative conclusion about what one should do/ought to do, or what should be the case. It's not exactly clear how the word is functioning here, but for (C) to be a contender, it would be helpful if there were a similar shift in the stimulus from descriptive statements to a normative "should" conclusion.

Get the most out of your LSAT Prep Plus subscription.

Analyze and track your performance with our Testing and Analytics Package.